From d9ca730a23d404d40e806baae7822e9da0f7e6db Mon Sep 17 00:00:00 2001 From: Adrian Kummerlaender Date: Sun, 12 Feb 2017 20:03:44 +0100 Subject: Introduce field aliases --- analysis.tex | 178 ++++++++++++++++++++++++++-------------------------- analysis_3.tex | 38 +++++------ lineare_algebra.tex | 44 ++++++------- numerik_1.tex | 24 +++---- zusammenfassung.tex | 4 ++ 5 files changed, 146 insertions(+), 142 deletions(-) diff --git a/analysis.tex b/analysis.tex index 65a98e9..f2e3edb 100644 --- a/analysis.tex +++ b/analysis.tex @@ -1,10 +1,10 @@ \section*{Folgen} -$\displaystyle\lim_{n \to \infty} = a \Leftrightarrow \forall \epsilon > 0 \exists N_\epsilon \in \mathbb{N} \forall n \geq N_\epsilon : | a_n - a | \leq \epsilon$ +$\displaystyle\lim_{n \to \infty} = a \Leftrightarrow \forall \epsilon > 0 \exists N_\epsilon \in \N \forall n \geq N_\epsilon : | a_n - a | \leq \epsilon$ \subsection*{Konvergenzsatz für monotone Folgen} -Sei $(a_n)$ wachsend und nach oben beschränkt \\ $\Rightarrow \exists \lim_{n \to \infty} a_n = \sup_{n\geq 1} a_n := \sup\{a_n | n \in \mathbb{N}\}$ +Sei $(a_n)$ wachsend und nach oben beschränkt \\ $\Rightarrow \exists \lim_{n \to \infty} a_n = \sup_{n\geq 1} a_n := \sup\{a_n | n \in \N\}$ Analoges für fallende, nach unten beschr. Folgen. @@ -18,15 +18,15 @@ $\underline\lim_{x \to \infty} a_n$ Minimum der Häufungspunkte \subsection*{Cauchyfolgen} -$\forall \epsilon > 0 \exists N_\epsilon \in \mathbb{N} \forall n, m \geq N_\epsilon : | a_n - a_m | \leq \epsilon$ +$\forall \epsilon > 0 \exists N_\epsilon \in \N \forall n, m \geq N_\epsilon : | a_n - a_m | \leq \epsilon$ Cauchyfolge $\Leftrightarrow$ konvergente Folge \subsection*{Uneigentliche Grenzwerte} -Sei $\overline{\mathbb{R}} = \mathbb{R} \cup \{-\infty, +\infty\}$ +Sei $\overline{\R} = \R \cup \{-\infty, +\infty\}$ -$\forall K \in \mathbb{N} \exists N_K \in \mathbb{N} \forall n \geq N_K : x_n \geq K \Leftrightarrow \displaystyle\lim_{n \to \infty} x_n = \infty$ +$\forall K \in \N \exists N_K \in \N \forall n \geq N_K : x_n \geq K \Leftrightarrow \displaystyle\lim_{n \to \infty} x_n = \infty$ \subsection*{Beispiele und Hinweise} @@ -46,20 +46,20 @@ $\sum_{k\geq 0} a_k$ konvergiert $\Rightarrow (a_k)$ ist Nullfolge. \subsubsection*{Leibnizkriterium} -Es gelte $\forall k \in \mathbb{N}_0 : b_k \geq b_{k+1} \geq 0$ und $\displaystyle \lim_{k \to \infty} b_k = 0$ +Es gelte $\forall k \in \N_0 : b_k \geq b_{k+1} \geq 0$ und $\displaystyle \lim_{k \to \infty} b_k = 0$ Dann konvergiert: $\sum_{k=0}^\infty (-1)^k b_k$ \subsubsection*{Majorantenkriterium} \begin{enumerate}[label=(\alph*)] - \item Wenn $0 \leq |a_k| \leq b_k \forall k \in \mathbb{N}_0$ und $\sum_k b_k$ konvergiert, dann konvergiert $\sum_k a_k$ absolut und es gilt $|\sum_{k=0}^\infty a_k| \leq \sum_{k=0}^\infty |a_k| \leq \sum_{k=0}^\infty b_k$. + \item Wenn $0 \leq |a_k| \leq b_k \forall k \in \N_0$ und $\sum_k b_k$ konvergiert, dann konvergiert $\sum_k a_k$ absolut und es gilt $|\sum_{k=0}^\infty a_k| \leq \sum_{k=0}^\infty |a_k| \leq \sum_{k=0}^\infty b_k$. \item Wenn $a_k \geq b_k \geq 0$ und $\sum_k b_k$ divergiert, dann divergiert $\sum_k a_k$ \end{enumerate} \subsubsection*{Quotientenkriterium} -Sei $(a_n)_{n\geq 0}$ Folge, $n_0 \in \mathbb{N}$ mit $\forall n \geq n_0 : a_n \neq 0$ +Sei $(a_n)_{n\geq 0}$ Folge, $n_0 \in \N$ mit $\forall n \geq n_0 : a_n \neq 0$ $\overline\lim_{n \to \infty} |\frac{a_{n+1}}{a_n}| < 1 \implies \sum a_n$ konvergiert absolut @@ -96,7 +96,7 @@ Die allgemeine harmonische Reihe $\sum_{k=0}^\infty \frac{1}{k^\alpha}$ divergie Eine Reihe $\sum_k a_k$ konvergiert genau dann, wenn: -$\forall \epsilon > 0 \exists N_\epsilon \in \mathbb{N} \forall n > m \geq N_\epsilon : | \sum_{k=m+1}^{n} a_k | \leq \epsilon$ +$\forall \epsilon > 0 \exists N_\epsilon \in \N \forall n > m \geq N_\epsilon : | \sum_{k=m+1}^{n} a_k | \leq \epsilon$ \section*{Potenzreihen} @@ -148,21 +148,21 @@ Wenn $z_0 \in D$ ein isolierter Punkt, dann ist $f$ in $z_0$ immer stetig. \subsection*{Abgeschlossenheit} -$D \subseteq \mathbb{C}$ ist abgeschlossen, wenn $z_n \in D \text{ für } n \in \mathbb{N} \land \lim_{n \to \infty} z_n = z \implies z \in D$. +$D \subseteq \mathbb{C}$ ist abgeschlossen, wenn $z_n \in D \text{ für } n \in \N \land \lim_{n \to \infty} z_n = z \implies z \in D$. \subsection*{Gleichmäßige Stetigkeit} -Sei $D \subseteq \mathbb{R}, f : D \rightarrow \mathbb{R}$. $f$ ist glm. stetig gdw.: +Sei $D \subseteq \R, f : D \rightarrow \R$. $f$ ist glm. stetig gdw.: $\forall \epsilon > 0 \exists \delta > 0 \forall x, x_0 \in D : | x - x_0 | < \delta \\ \hspace*{4mm} \implies | f(x) - f(x_0) | < \epsilon$ \subsection*{Satz vom Maximum} -Sei $D \subseteq \mathbb{C}$ abgeschlossen und beschränkt, $f: D \rightarrow \mathbb{R}$ stetig, dann nimmt $f$ auf $D$ ein Maximum und Minimum an, ist also insbesondere beschränkt. +Sei $D \subseteq \mathbb{C}$ abgeschlossen und beschränkt, $f: D \rightarrow \R$ stetig, dann nimmt $f$ auf $D$ ein Maximum und Minimum an, ist also insbesondere beschränkt. \subsection*{Zwischenwertsatz} -Sei $f: [a, b] \rightarrow \mathbb{R}$ stetig, dann: +Sei $f: [a, b] \rightarrow \R$ stetig, dann: $f([a, b]) = [\min_{[a, b]} f, \max_{[a, b]} f]$. @@ -172,13 +172,13 @@ $\forall y \in [\min_{[a, b]} f, \max_{[a, b]} f] \exists x \in [a, b]: f(x)=y$ \subsection*{Nullstellensatz} -Sei $f \in [a, b] \rightarrow \mathbb{R}$ stetig mit $f(a)f(b) \leq 0$, dann: +Sei $f \in [a, b] \rightarrow \R$ stetig mit $f(a)f(b) \leq 0$, dann: $\exists x \in [a, b]: f(x) = 0$ \subsection*{Intervallsatz} -Sei $I \subseteq \mathbb{R}$ ein Intervall und $f : I \rightarrow \mathbb{R}$ stetig. +Sei $I \subseteq \R$ ein Intervall und $f : I \rightarrow \R$ stetig. Dann ist $f(I)$ ein Intervall. @@ -188,7 +188,7 @@ Dann ist $f(I)$ ein Intervall. Fkt. Folge $(f_n)$ konv. punktweise gegen $f$, wenn: -$\forall z \in D \forall \epsilon > 0 \exists N_{\epsilon, z} \in \mathbb{N} \forall n \geq N_{\epsilon, z} : | f_n(z) - f(z) | \leq \epsilon$ +$\forall z \in D \forall \epsilon > 0 \exists N_{\epsilon, z} \in \N \forall n \geq N_{\epsilon, z} : | f_n(z) - f(z) | \leq \epsilon$ Dies ist äquivalent zu $\lim_{n\to \infty} f_n(x)=f(x)$. @@ -196,7 +196,7 @@ Dies ist äquivalent zu $\lim_{n\to \infty} f_n(x)=f(x)$. Fkt. Folge $(f_n)$ konv. gleichmäßig gegen $f$, wenn: -$\forall z \in D \exists N_\epsilon \in \mathbb{N} \forall n \geq N_\epsilon : \sup_{z \in D} | f_n(z) - f(z) | \leq \epsilon$ +$\forall z \in D \exists N_\epsilon \in \N \forall n \geq N_\epsilon : \sup_{z \in D} | f_n(z) - f(z) | \leq \epsilon$ Dies ist äquivalent zu $\displaystyle\lim_{n\to \infty}(\sup_{z \in D}| f_n(z) - f(z) |) = 0$. @@ -204,7 +204,7 @@ Dies ist äquivalent zu $\displaystyle\lim_{n\to \infty}(\sup_{z \in D}| f_n(z) \subsection*{Differenzierbarkeit} -Funktion $f : I \rightarrow \mathbb{R}$ ist in $x_0$ differenzierbar, wenn: +Funktion $f : I \rightarrow \R$ ist in $x_0$ differenzierbar, wenn: $\lim_{x \to x_0} \frac{f(x) - f(x_0)}{x - x_0} =: f'(x_0) = \frac{df}{dx}(x_0)$ existiert. @@ -245,7 +245,7 @@ $\lim_{x \to x_0} \frac{f(x) - f(x_0)}{x - x_0} =: f'(x_0) = \frac{df}{dx}(x_0)$ \subsection*{Mittelwertsatz} -Sei $a < b$ in $\mathbb{R}$ und $f, g \in C^1((a, b),\mathbb{R})$, dann: +Sei $a < b$ in $\R$ und $f, g \in C^1((a, b),\R)$, dann: $\exists \xi \in (a, b) : ( f(b) - f(a) ) g'(\xi) = f'(\xi)(g(b) - g(a))$ @@ -273,20 +273,20 @@ Seien $x, y > 0$, $p \in (1, \infty)$ und $p' := \frac{p}{p-1}$, also $\frac{1}{ \subsection*{L'Hospital} -Sei $-\infty \leq a < b \leq +\infty$, $f, g : (a, b) \rightarrow \mathbb{R}$ differenzierbar mit $\forall x \in (a, b) : g'(x) \neq 0$ und es gelte eines: +Sei $-\infty \leq a < b \leq +\infty$, $f, g : (a, b) \rightarrow \R$ differenzierbar mit $\forall x \in (a, b) : g'(x) \neq 0$ und es gelte eines: \begin{enumerate}[label=(\alph*)] \item $\exists \lim_{x \to b^-} f(x) = \lim_{x \to b^-} g(x) = 0$ \item $\exists \lim_{x \to b^-} g(x) = \pm \infty$ \end{enumerate} -Ferner existiere $l \in \overline{\mathbb{R}}$ mit $\lim_{x \to b^-} \frac{f'(x)}{g'(x)} = l$. +Ferner existiere $l \in \overline{\R}$ mit $\lim_{x \to b^-} \frac{f'(x)}{g'(x)} = l$. Dann existiert $\lim_{x \to b^-} \frac{f(x)}{g(x)} = l$. \subsection*{Umkehrregel} -Sei $f : I \subseteq \mathbb{R} \to \mathbb{R}$ strikt monoton, stetig und differenzierbar in $x_0 \in I$ mit $f'(x_0) \neq 0$. Dann ist $f^{-1} : f(I) \to \mathbb{R}$ in $y_0 = f(x_0)$ differenzierbar mit: +Sei $f : I \subseteq \R \to \R$ strikt monoton, stetig und differenzierbar in $x_0 \in I$ mit $f'(x_0) \neq 0$. Dann ist $f^{-1} : f(I) \to \R$ in $y_0 = f(x_0)$ differenzierbar mit: $(f^{-1})'(y_0) = \frac{1}{f'(f^{-1}(y_0))} = \frac{1}{f'(x_0)}$ @@ -308,7 +308,7 @@ $R_{n-1, x_0} f(x) = \frac{1}{n!} f^{(n)} (x_0 + \theta(x-x_0))(x-x_0)^n$ \subsection*{Integralrestglied} -Sei $f \in C^{n+1}((a, b)), n \in \mathbb{N}_0, x_0 \in (a, b)$, dann gilt: +Sei $f \in C^{n+1}((a, b)), n \in \N_0, x_0 \in (a, b)$, dann gilt: $f(x) - T_{n,x_0} f(x) = \frac{1}{n!} \int_{x_0}^x (x-t)^n f^{(n+1)}(t) dt$ @@ -316,7 +316,7 @@ $f(x) - T_{n,x_0} f(x) = \frac{1}{n!} \int_{x_0}^x (x-t)^n f^{(n+1)}(t) dt$ \subsection*{Stückweise Stetigkeit} -$f : [a, b] \rightarrow \mathbb{R}$ ist stückweise stetig, wenn sich $[a, b]$ in endlich viele disjunkte Teilintervalle so aufteilen lässt, dass $f$ eingeschränkt auf diese stetig ist. +$f : [a, b] \rightarrow \R$ ist stückweise stetig, wenn sich $[a, b]$ in endlich viele disjunkte Teilintervalle so aufteilen lässt, dass $f$ eingeschränkt auf diese stetig ist. $PC([a, b])$ ist Menge stckw. stg. Fkt. über $[a, b]$. @@ -340,13 +340,13 @@ $\int_a^b f(\phi(x))\phi'(x) dx = \int_{\phi(a)}^{\phi(b)} f(y) dy$ \subsection*{Uneigentliche Riemann-Integrale} -Sei $-\infty < a < b \leq +\infty, f : [a, b) \rightarrow \mathbb{R}$ so, dass $\forall \beta \in (a, b): f|_{[a, \beta]} \in PC([a, \beta])$. +Sei $-\infty < a < b \leq +\infty, f : [a, b) \rightarrow \R$ so, dass $\forall \beta \in (a, b): f|_{[a, \beta]} \in PC([a, \beta])$. -Falls $\lim_{\beta \to b^-} \int_a^\beta f(x) dx =: \int_a^b f(x) dx$ in $\mathbb{R}$ existiert, heißt $f$ uneigentlich Riemann-integrierbar. +Falls $\lim_{\beta \to b^-} \int_a^\beta f(x) dx =: \int_a^b f(x) dx$ in $\R$ existiert, heißt $f$ uneigentlich Riemann-integrierbar. \subsubsection*{Majoranten- / Minorantenkriterium} -Sei $-\infty < a < b \leq +\infty$ und $f, g : [a, b) \to \mathbb{R}$ mit $\forall c \in (a, b) : f, g \in PC([a, c])$, dann: +Sei $-\infty < a < b \leq +\infty$ und $f, g : [a, b) \to \R$ mit $\forall c \in (a, b) : f, g \in PC([a, c])$, dann: \begin{enumerate}[label=(\alph*)] \item $\forall x \in [a, b) : |f(x)| \leq g(x) \land g$ uneigentlich integrierbar $\Rightarrow$ $f, |f|$ uneigentlich integrierbar. @@ -388,7 +388,7 @@ Sei $-\infty < a < b \leq +\infty$ und $f, g : [a, b) \to \mathbb{R}$ mit $\fora Folge $(x_n) \subseteq X$ konvergiert gegen $x \in X$, wenn: -$\forall \epsilon > 0 \exists N_\epsilon \in \mathbb{N} \forall n \geq N_\epsilon : ||x_n - x|| \leq \epsilon$ +$\forall \epsilon > 0 \exists N_\epsilon \in \N \forall n \geq N_\epsilon : ||x_n - x|| \leq \epsilon$ \subsubsection*{$p$-Norm} @@ -399,13 +399,13 @@ $$|x|_p := \begin{cases} \subsubsection*{Hölder-Ungleichung} -Sei $p \in |1, \infty]$ mit $\frac{1}{p}+\frac{1}{p'}=1$ sowie $x, y \in \mathbb{K}^m$: +Sei $p \in |1, \infty]$ mit $\frac{1}{p}+\frac{1}{p'}=1$ sowie $x, y \in \K^m$: $|\sum_{k=1}^m x_k y_k| \leq \sum_{k=1}^m |x_k y_k| \leq |x|_p |x|_{p'}$ \subsubsection*{Minkowski-Ungleichung} -Sei $x, y \in \mathbb{K}^m$: $|x+y|_p \leq |x|_p + |y|_p$ +Sei $x, y \in \K^m$: $|x+y|_p \leq |x|_p + |y|_p$ \subsubsection*{Cauchy-Schwarz-Ungleichung} @@ -427,7 +427,7 @@ $||\cdot|| \sim |||\cdot||| \Leftrightarrow \exists C, c > 0 \forall x \in X \la \subsubsection*{Konvergenz bezüglich $p$-Normen} -Sei $1 \leq p < q \leq \infty$, $x \in \mathbb{K}^m$, dann gilt: +Sei $1 \leq p < q \leq \infty$, $x \in \K^m$, dann gilt: $|x|_p \leq m^{\frac{1}{p} - \frac{1}{q}} |x|_q$ und $|x|_q \leq |x|_p$ @@ -435,7 +435,7 @@ $|x|_p \leq m^{\frac{1}{p} - \frac{1}{q}} |x|_q$ und $|x|_q \leq |x|_p$ Sei $(X, ||\cdot||)$ normierter Vektorraum. $(x_n) \subseteq X$ ist Cauchyfolge, wenn gilt: -$\forall \epsilon > 0 \exists N_\epsilon \in \mathbb{N} \forall n, m \geq N_\epsilon : ||x_n - x_m|| \leq \epsilon$ +$\forall \epsilon > 0 \exists N_\epsilon \in \N \forall n, m \geq N_\epsilon : ||x_n - x_m|| \leq \epsilon$ \subsection*{Normen stetiger Funktionen} @@ -454,9 +454,9 @@ Wenn jede Cauchyfolge in $(X, ||\cdot||)$ einen Grenzwert besitzt, dann heißt $ \subsubsection*{Hilbertraum} -Die Norm des Banachraums $(\mathbb{K}^m, |\cdot|_2)$ ist durch Skalarprodukt gegeben. Ein solcher Banachraum heißt Hilbertraum. +Die Norm des Banachraums $(\K^m, |\cdot|_2)$ ist durch Skalarprodukt gegeben. Ein solcher Banachraum heißt Hilbertraum. -Ein $\mathbb{R}$-VRaum mit SKP ist ein euklidischer Raum. +Ein $\R$-VRaum mit SKP ist ein euklidischer Raum. \subsubsection*{Äquivalenz der Normen} @@ -474,13 +474,13 @@ Sei $V$ endlichdim. VRaum, dann sind alle Normen auf $V$ äquivalent. Insb. ist $(x_n) \subseteq X$ konvergiert in $(M, d)$ gegen $x \in X$, wenn: -$\forall \epsilon > 0 \exists N_\epsilon \in \mathbb{N} \forall n \geq N_\epsilon : d(x_n, x) \leq \epsilon$ +$\forall \epsilon > 0 \exists N_\epsilon \in \N \forall n \geq N_\epsilon : d(x_n, x) \leq \epsilon$ \subsubsection*{Cauchyfolgen bzgl. Metriken} Eine Folge $(x_n) \subseteq M$ heißt Cauchfolge, wenn: -$\forall \epsilon > 0 \exists N_\epsilon \in \mathbb{N} \forall n, m \geq N_\epsilon : d(x_n, x_m) \leq \epsilon$ +$\forall \epsilon > 0 \exists N_\epsilon \in \N \forall n, m \geq N_\epsilon : d(x_n, x_m) \leq \epsilon$ Ein metrischer Raum ist vollständig, wenn jede Cauchfolge in $(M, d)$ konvergiert. @@ -560,7 +560,7 @@ $\forall x, y \in M : d_N(f(x), f(y)) \leq L d_M(x, y)$ Lipschitz stetige Fkt. sind insb. auch glm. stetig. -$L(V, W) = \{T : V \rightarrow W | T \text{ linear} \}$ sind lineare Abb. zwischen $\mathbb{K}$-VR $V$ und $W$, genannt Operatoren. +$L(V, W) = \{T : V \rightarrow W | T \text{ linear} \}$ sind lineare Abb. zwischen $\K$-VR $V$ und $W$, genannt Operatoren. \subsubsection*{Operatornorm} @@ -594,7 +594,7 @@ $f$ bijektiv $\Rightarrow f$ ist homömorph, d.h. bijektiv, stetig und hat steti \subsubsection*{Satz vom Maximum} -Sei $K$ komp. metr. Raum, $f: K \rightarrow \mathbb{R}$ stetig, dann: +Sei $K$ komp. metr. Raum, $f: K \rightarrow \R$ stetig, dann: $\exists x_{\pm} \in K : f(x_+) = \displaystyle\max_{x \in K} f(x) \land f(x_-) = \displaystyle\min_{x \in K} f(x)$ @@ -606,19 +606,19 @@ $\forall x, y \in M \exists w \in C([0,1], M) : w(0) = x \land w(1) = y$ \subsubsection*{Zwischenwertsatz} -Seien $M$, $N$ metr. Räume, $M$ wegzusammenhängend und $f \in C(M, N)$. Dann ist $f(M)$ wegzusammenhängend. Für $N = \mathbb{R}$ ist $f(M)$ ein Intervall. +Seien $M$, $N$ metr. Räume, $M$ wegzusammenhängend und $f \in C(M, N)$. Dann ist $f(M)$ wegzusammenhängend. Für $N = \R$ ist $f(M)$ ein Intervall. \section*{Differentialrechnung in VRäumen} \subsection*{Kurventangente} -Sei $J$ ein Intervall. Ein Weg ist stetige Abbildung $f = (f_1 \hdots f_m)^T : J \rightarrow \mathbb{R}^m$. Das Bild $\Gamma = f(J)$ heißt Kurve. $f$ ist Parametrisierung von $\Gamma$. +Sei $J$ ein Intervall. Ein Weg ist stetige Abbildung $f = (f_1 \hdots f_m)^T : J \rightarrow \R^m$. Das Bild $\Gamma = f(J)$ heißt Kurve. $f$ ist Parametrisierung von $\Gamma$. Die Ableitung eines $C^1$-Wegs $f$ in $t_0 \in J$ ist def.: -$f'(t_0) = \displaystyle\lim_{h \to 0} \frac{1}{h} (f(t_0 + h) - f(t_0)) = \begin{pmatrix} f_1'(t_0) \\ \vdots \\ f_m'(t_0)\end{pmatrix} \in \mathbb{R}^m$ +$f'(t_0) = \displaystyle\lim_{h \to 0} \frac{1}{h} (f(t_0 + h) - f(t_0)) = \begin{pmatrix} f_1'(t_0) \\ \vdots \\ f_m'(t_0)\end{pmatrix} \in \R^m$ -Für $f'(t_0) \neq 0$ ist Tangente $T(\mathbb{R})$ an $f(t_0)$ durch $T(t) = f(t_0) + (t - t_0)f'(t_0)$ mit $t \in \mathbb{R}$ gegeben. +Für $f'(t_0) \neq 0$ ist Tangente $T(\R)$ an $f(t_0)$ durch $T(t) = f(t_0) + (t - t_0)f'(t_0)$ mit $t \in \R$ gegeben. \subsection*{Partielle Ableitungen} @@ -634,31 +634,31 @@ $f'(x_0) := A$ ist Ableitung von $f$ bei $x_0$. Wenn $\forall x_0 \in D : f $ di \subsubsection*{Jacobimatrix} -Sei $D \subseteq \mathbb{R}^l$ offen, $f : D \rightarrow \mathbb{R}^k$, $i \in \{1, ..., k\}$, $j \in \{1, ..., l\}$ und $f = (f_1 \hdots f_m)^T$, dann: +Sei $D \subseteq \R^l$ offen, $f : D \rightarrow \R^k$, $i \in \{1, ..., k\}$, $j \in \{1, ..., l\}$ und $f = (f_1 \hdots f_m)^T$, dann: $\partial f(x) = \begin{pmatrix} \partial_1 f_1(x) & \hdots & \partial_l f_1(x) \\ \vdots & & \vdots \\ \partial_1 f_k(x) & \hdots & \partial_l f_k(x) \end{pmatrix}$ \subsubsection*{Gradient} -Wenn $D \subseteq \mathbb{R}^m$ offen und $f : D \rightarrow \mathbb{R}$ bei $x \in D$ differenzierbar ist, dann: +Wenn $D \subseteq \R^m$ offen und $f : D \rightarrow \R$ bei $x \in D$ differenzierbar ist, dann: -$\nabla f(x) := \begin{pmatrix} \partial_1 f(x) \\ \vdots \\ \partial_m f(x) \end{pmatrix} = f'(x)^T \in \mathbb{R}^m$ +$\nabla f(x) := \begin{pmatrix} \partial_1 f(x) \\ \vdots \\ \partial_m f(x) \end{pmatrix} = f'(x)^T \in \R^m$ Identifiziert kritische Stellen mit $\nabla f(x) = 0$. \subsection*{Richtungsableitung} -Sei $D \subseteq \mathbb{R}^m$ offen, $f : D \rightarrow \mathbb{R}$ und $v \in \mathbb{R}^m \setminus \{0\}$, dann ist Ableitung von $f$ bei $x$ in Richtung $v$: +Sei $D \subseteq \R^m$ offen, $f : D \rightarrow \R$ und $v \in \R^m \setminus \{0\}$, dann ist Ableitung von $f$ bei $x$ in Richtung $v$: $\partial_v f(x) = \frac{\partial f}{\partial v}(x) := \displaystyle\lim_{t \to 0} \frac{1}{t} (f(x+tv)-f(x))$ -Insofern Grenzwert in $\mathbb{R}$ existiert. Weiterhin gilt: +Insofern Grenzwert in $\R$ existiert. Weiterhin gilt: -$\partial_v f(x) = (\nabla f(x) | v)$ für $f \in C^1(D, \mathbb{R})$. +$\partial_v f(x) = (\nabla f(x) | v)$ für $f \in C^1(D, \R)$. \subsubsection*{Hessematrix} -Wenn $D \subseteq \mathbb{R}^m$ offen und $f \in C^n(D, \mathbb{R})$, dann $\nabla f \in C^1(D,\mathbb{R}^m)$ und somit: +Wenn $D \subseteq \R^m$ offen und $f \in C^n(D, \R)$, dann $\nabla f \in C^1(D,\R^m)$ und somit: $\nabla^2 f(x) = \begin{pmatrix} \partial_1 \partial_1 f(x) & \hdots & \partial_m \partial_1 f(x) \\ \vdots & & \vdots \\ \partial_1 \partial_m f(x) & \hdots & \partial_m \partial_m f(x) \end{pmatrix}$ @@ -666,7 +666,7 @@ $\nabla^2 f(x) = \begin{pmatrix} \partial_1 \partial_1 f(x) & \hdots & \partial_ \subsection*{Taylor in mehreren Dimensionen} -Sei $D \subseteq \mathbb{R}^l$ offen, $f \in C^{n+1}(D, \mathbb{R})$, $x \in D$, $r > 0$ mit $\overline B(x,r) \subseteq D$ und $h \in \mathbb{R}^l$ mit $|h|_2 < r$, dann: +Sei $D \subseteq \R^l$ offen, $f \in C^{n+1}(D, \R)$, $x \in D$, $r > 0$ mit $\overline B(x,r) \subseteq D$ und $h \in \R^l$ mit $|h|_2 < r$, dann: $T_{n,x} f(x+h) = f(x) + \displaystyle\sum_{j=1}^n \frac{1}{j!} \displaystyle\sum_{\alpha_1, ..., \alpha_j=1}^l h_{\alpha_1} \cdot \hdots \cdot h_{\alpha_j} ( \partial_{\alpha_1} \hdots \partial_{\alpha_j} f )(x)$ @@ -680,18 +680,18 @@ $R_{n,x} f(x+h) = f(x+h) - T_{n,x}f(x+h)$ \subsection*{Definitheit} -Sei $A \in L(\mathbb{R}^m)$ symmetrisch, dann ist $A$: +Sei $A \in L(\R^m)$ symmetrisch, dann ist $A$: \begin{description}[leftmargin=!,labelwidth=28mm] - \item[positiv definit] $\forall v \in \mathbb{R}^m \setminus \{0\} : (Av|v) > 0$ - \item[negativ definit] $\forall v \in \mathbb{R}^m \setminus \{0\} : (Av|v) > 0$ - \item[positiv semidefinit] $\forall v \in \mathbb{R}^m : (Av|v) \geq 0$ - \item[negativ semidefinit] $\forall v \in \mathbb{R}^m : (Av|v) \leq 0$ + \item[positiv definit] $\forall v \in \R^m \setminus \{0\} : (Av|v) > 0$ + \item[negativ definit] $\forall v \in \R^m \setminus \{0\} : (Av|v) > 0$ + \item[positiv semidefinit] $\forall v \in \R^m : (Av|v) \geq 0$ + \item[negativ semidefinit] $\forall v \in \R^m : (Av|v) \leq 0$ \end{description} \subsubsection*{Definitheit von $2\times 2$ Hessematrizen} -Sei $A = \begin{pmatrix} a & b \\ b & d \end{pmatrix} \in \mathbb{R}^{2 \times 2}$ eine symmetrische Matrix: +Sei $A = \begin{pmatrix} a & b \\ b & d \end{pmatrix} \in \R^{2 \times 2}$ eine symmetrische Matrix: \begin{enumerate}[label=(\alph*)] \item positiv definit $\Leftrightarrow$ $a > 0, ad > b^2$ @@ -712,7 +712,7 @@ Sei $A = \begin{pmatrix} a & b \\ b & d \end{pmatrix} \in \mathbb{R}^{2 \times 2 \subsection*{Extremstellen} -Seien $D \subseteq \mathbb{R}^m$ offen, $f \in C^2(D, \mathbb{R})$, $x \in D$, dann: +Seien $D \subseteq \R^m$ offen, $f \in C^2(D, \R)$, $x \in D$, dann: \begin{description}[leftmargin=!,labelwidth=28mm] \item[Maximum] $\nabla^2 f(x)$ negativ semidefinit @@ -728,26 +728,26 @@ Wenn $\nabla f(x) = 0$: \subsection*{Umkehrsatz} -Seien $D \subseteq \mathbb{R}^m$ offen, $f \in C^1(D, \mathbb{R}^m)$, $x_0 \in D$, $y_0 = f(x_0)$, $f'(x_0) \in L(\mathbb{R}^m$ bijektiv, dann: +Seien $D \subseteq \R^m$ offen, $f \in C^1(D, \R^m)$, $x_0 \in D$, $y_0 = f(x_0)$, $f'(x_0) \in L(\R^m$ bijektiv, dann: -$\exists U \subseteq D \text{ offen }, V \subseteq \mathbb{R}^m : x_0 \in U, y_0 \in V, f_U : U \rightarrow V \text{ bijektiv }, V \subseteq f(D), (f_U)^{-1} \in C^1(V, U), \forall x \in U : f'(x) \text{ invertierbar}$. Insbesondere: +$\exists U \subseteq D \text{ offen }, V \subseteq \R^m : x_0 \in U, y_0 \in V, f_U : U \rightarrow V \text{ bijektiv }, V \subseteq f(D), (f_U)^{-1} \in C^1(V, U), \forall x \in U : f'(x) \text{ invertierbar}$. Insbesondere: $\forall y=f(x) \in V : (f_U^{-1})'(y) = f'(f_U^{-1}(y))^{-1} = f'(x)^{-1}$ \subsubsection*{Diffeomorphismen} -Seien $D \subseteq \mathbb{R}^m$ offen, $f \in C^1(D, \mathbb{R}^m)$, $\tilde D = f(D)$, $f$ injektiv, $\forall x \in D: f'(x)$ invertierbar. Dann ist $\tilde D$ offen und $f : D \rightarrow \tilde D$ ein Diffeomorphismus, d.h. $D$, $\tilde D$ offen, $f$ bijektiv und $f \in C^1(D, \mathbb{R}^m)$, $f^{-1} \in C^1(\tilde D, \mathbb{R}^m)$ +Seien $D \subseteq \R^m$ offen, $f \in C^1(D, \R^m)$, $\tilde D = f(D)$, $f$ injektiv, $\forall x \in D: f'(x)$ invertierbar. Dann ist $\tilde D$ offen und $f : D \rightarrow \tilde D$ ein Diffeomorphismus, d.h. $D$, $\tilde D$ offen, $f$ bijektiv und $f \in C^1(D, \R^m)$, $f^{-1} \in C^1(\tilde D, \R^m)$ \subsubsection*{Polarkoordinaten} -$\phi : D = (\mathbb{R} \setminus \{0\}) \times \mathbb{R} \rightarrow \mathbb{R}^2; (r, \varphi) \mapsto \begin{pmatrix} r \cos \varphi \\ r \sin \varphi \end{pmatrix} = \begin{pmatrix} x \\ y \end{pmatrix}$ +$\phi : D = (\R \setminus \{0\}) \times \R \rightarrow \R^2; (r, \varphi) \mapsto \begin{pmatrix} r \cos \varphi \\ r \sin \varphi \end{pmatrix} = \begin{pmatrix} x \\ y \end{pmatrix}$ \subsection*{Satz über implizit definierte Funktionen} -Seien $D \subseteq \mathbb{R}^{m+k}$ offen, $f \in C^1(D, \mathbb{R}^k)$, $(x_0, y_0) \in D$ mit $f(x_0, y_0) = 0$ und $(\partial_y f)(x_0, y_0) \in L(\mathbb{R}^k)$ bijektiv. Dann $\exists \text{ offene } U_x \subseteq \mathbb{R}^m, U_y \subseteq \mathbb{R}^k$ sowie Abbildung $\varphi \in C^1(U, \mathbb{R}^k)$ mit: +Seien $D \subseteq \R^{m+k}$ offen, $f \in C^1(D, \R^k)$, $(x_0, y_0) \in D$ mit $f(x_0, y_0) = 0$ und $(\partial_y f)(x_0, y_0) \in L(\R^k)$ bijektiv. Dann $\exists \text{ offene } U_x \subseteq \R^m, U_y \subseteq \R^k$ sowie Abbildung $\varphi \in C^1(U, \R^k)$ mit: \begin{enumerate}[label=(\alph*)] - \item $(x_0, y_0) \in U_x \times U_y \subseteq D$, $\varphi(U_x) \subseteq U_y$, $\varphi(x_0)=y_0$, $\forall (x, y) \in U_x \times U_y : \partial_y f(x,y) \in L(\mathbb{R}^k)$ bijektiv + \item $(x_0, y_0) \in U_x \times U_y \subseteq D$, $\varphi(U_x) \subseteq U_y$, $\varphi(x_0)=y_0$, $\forall (x, y) \in U_x \times U_y : \partial_y f(x,y) \in L(\R^k)$ bijektiv \item $(x, y) \in U_x \times U_y \land f(x,y)=0 \\ \Leftrightarrow x \in U_x \land y=\varphi(x)$ \item $\varphi'(x) = -[(\partial_y f)(x, \varphi(x))]^{-1}(\partial_x f)(x, \varphi(x))$ \end{enumerate} @@ -756,32 +756,32 @@ Insbesondere auch $\forall x \in U_x : f(x, \varphi(x))=0$ \subsection*{$C^1$-Flächen} -$M \subseteq \mathbb{R}^3$ ist eingebettete $C^1$-Fläche, wenn $\forall p \in M \exists \text{ offene } V, U \subseteq \mathbb{R}^3 : p \in V \land \psi : V \rightarrow U$ so, dass $\psi(V \cap M) = U \cap (\mathbb{R}^2 \times \{0\})$. $\psi$ heißt dann Karte. +$M \subseteq \R^3$ ist eingebettete $C^1$-Fläche, wenn $\forall p \in M \exists \text{ offene } V, U \subseteq \R^3 : p \in V \land \psi : V \rightarrow U$ so, dass $\psi(V \cap M) = U \cap (\R^2 \times \{0\})$. $\psi$ heißt dann Karte. \subsubsection*{Charakterisierung} \begin{enumerate}[label=(\alph*)] \item $M$ ist $C^1$-Fläche - \item $\forall p \in M \exists \text{ offenes } D \subseteq \mathbb{R}^3 \land g \in C^1(D, \mathbb{R}) : \\ p \in D \land \forall w \in D : \nabla g(w) \neq 0 \land M \cap D = \{w\in D | g(w) = 0\}$ (lokale Nullstellenmenge) - \item $\forall p \in M \exists i < j \in \{1, 2, 3\}, \text{ offene } U_1 \subseteq \mathbb{R}^1, U_2 \subseteq \mathbb{R}^2, h \in C^1(U_2, U_1) : p_k \in U_1, (p_i, p_j) \in U_2$ mit $\{k\} = \{1, 2, 3\} \setminus \{j, j\}$, $h(U_2) \subseteq U_1$ und für $Z=\{x \in \mathbb{R}^3 | (x_i, x_j) \in U_2, x_k \in U_1\}$ gilt $M \cap Z = \{x \in \mathbb{R}^3 | (x_i, x_j) \in U_2, x_k = h(x_i, x_j)\}$ (lokaler Graph) - \item $\forall p \in M \exists \text{ offenes } U_0 \subseteq \mathbb{R}^2, W \subseteq \mathbb{R}^3, F \in C^1(U_0, \mathbb{R}^3) : p \in W, \forall (s, t) \in U_0 : Rg(F'(s, t)) = 2$ und $F: U_0 \rightarrow M \cap W$ bijektiv mit stetiger Umkehrabbildung. (lokale Parameterisierung) + \item $\forall p \in M \exists \text{ offenes } D \subseteq \R^3 \land g \in C^1(D, \R) : \\ p \in D \land \forall w \in D : \nabla g(w) \neq 0 \land M \cap D = \{w\in D | g(w) = 0\}$ (lokale Nullstellenmenge) + \item $\forall p \in M \exists i < j \in \{1, 2, 3\}, \text{ offene } U_1 \subseteq \R^1, U_2 \subseteq \R^2, h \in C^1(U_2, U_1) : p_k \in U_1, (p_i, p_j) \in U_2$ mit $\{k\} = \{1, 2, 3\} \setminus \{j, j\}$, $h(U_2) \subseteq U_1$ und für $Z=\{x \in \R^3 | (x_i, x_j) \in U_2, x_k \in U_1\}$ gilt $M \cap Z = \{x \in \R^3 | (x_i, x_j) \in U_2, x_k = h(x_i, x_j)\}$ (lokaler Graph) + \item $\forall p \in M \exists \text{ offenes } U_0 \subseteq \R^2, W \subseteq \R^3, F \in C^1(U_0, \R^3) : p \in W, \forall (s, t) \in U_0 : Rg(F'(s, t)) = 2$ und $F: U_0 \rightarrow M \cap W$ bijektiv mit stetiger Umkehrabbildung. (lokale Parameterisierung) \end{enumerate} \subsubsection*{Tangentialraum} Tangentialraum an $m$ bei $p=F(u_0)$ wobei $F$ lokale Parametrisierung: -$T_p M = F'(u_0)(\mathbb{R}^2) = \left\{ F'(u_0) \begin{pmatrix} s \\ t \end{pmatrix} \middle| s, t \in \mathbb{R} \right\}$ +$T_p M = F'(u_0)(\R^2) = \left\{ F'(u_0) \begin{pmatrix} s \\ t \end{pmatrix} \middle| s, t \in \R \right\}$ \subsubsection*{Normalenraum} Normalenraum an $M$ bei $p$ und $g(p)=0$ wobei $g$ lokale Nullstellenmenge: -$N_p M = lin\{\nabla g(p)\} = \{t\nabla g(p) | t \in \mathbb{R}\}$ +$N_p M = lin\{\nabla g(p)\} = \{t\nabla g(p) | t \in \R\}$ \subsection*{Lagrange} -Seien $D \subseteq \mathbb{R}^l$ offen, $f \in C^1(D, \mathbb{R}$ und $g \in C^1(D, \mathbb{R}^k)$ mit $1 \leq k \leq l$. Setze $M=\{z\in D| g(z)=0\}$. $f$ besitze auf $M$ ein Extremum bei $z_0 \in M$ und $g'(z_0)$ habe Rang $k$. Dann existieren Lagrangesche Multiplikatoren $\lambda_1, ..., \lambda_k \in \mathbb{R}$ so, dass: +Seien $D \subseteq \R^l$ offen, $f \in C^1(D, \R$ und $g \in C^1(D, \R^k)$ mit $1 \leq k \leq l$. Setze $M=\{z\in D| g(z)=0\}$. $f$ besitze auf $M$ ein Extremum bei $z_0 \in M$ und $g'(z_0)$ habe Rang $k$. Dann existieren Lagrangesche Multiplikatoren $\lambda_1, ..., \lambda_k \in \R$ so, dass: $\nabla f(z_0) = \lambda_1\nabla g_1(z_0) + \hdots + \lambda_k\nabla g_k(z_0)$ @@ -789,13 +789,13 @@ Ferner gilt $g_1(z_0) = 0, \hdots, g_k(z_0)=0$. \section*{Kurvenintegrale} -Sei $\gamma \in C([a, b], \mathbb{R}^m)$ ein Weg und $\Gamma = \gamma([a, b])$ die dazugehörige Kurve. Die Länge von $\Gamma$ wird durch Polygonzüge approximiert. Sei dazu $a \leq b$ und $\mathcal{Z}(a, b)$ die Menge aller Zerlegungen $Z = \{a = t_0, t_1, ..., t_{n-1}, t_n = b\}$. Für $Z \in \mathcal{Z}$ wird gesetzt: +Sei $\gamma \in C([a, b], \R^m)$ ein Weg und $\Gamma = \gamma([a, b])$ die dazugehörige Kurve. Die Länge von $\Gamma$ wird durch Polygonzüge approximiert. Sei dazu $a \leq b$ und $\mathcal{Z}(a, b)$ die Menge aller Zerlegungen $Z = \{a = t_0, t_1, ..., t_{n-1}, t_n = b\}$. Für $Z \in \mathcal{Z}$ wird gesetzt: $l(\gamma, Z) = \sum_{j=1}^n |\gamma(t_j) - \gamma(t_{j-1})|_2$ \subsection*{Rektifizierbarkeit} -$\gamma : [a, b] \rightarrow \mathbb{R}^m$ ist rektifizierbar, wenn: +$\gamma : [a, b] \rightarrow \R^m$ ist rektifizierbar, wenn: $l_{[a, b]}(\gamma) = ||\gamma||_{BV} := \sup_{Z\in \mathcal{Z}} l(\gamma, Z) < \infty$ @@ -803,35 +803,35 @@ $l(\gamma)$ ist dann die Länge von $\gamma$. \subsubsection*{Wegintegral} -Sei $\gamma \in C^1([a, b], \mathbb{R}^m)$, dann ist $\gamma$ rektifizierbar mit: +Sei $\gamma \in C^1([a, b], \R^m)$, dann ist $\gamma$ rektifizierbar mit: $l(\gamma) = \int_a^b |\gamma'(t)|_2 dt$ \subsection*{Kurvenintegrale und Potentiale} -Sei $\gamma \in C([a, b], \mathbb{R}^m)$ stückweise $C^1$, $\Gamma = \gamma([a, b])$. +Sei $\gamma \in C([a, b], \R^m)$ stückweise $C^1$, $\Gamma = \gamma([a, b])$. \subsubsection*{Kurvenintegral erster Art} -Sei reelles $f \in C(\Gamma, \mathbb{R})$ gegeben: +Sei reelles $f \in C(\Gamma, \R)$ gegeben: \vspace*{-5mm} $$\int_\Gamma f d\gamma = \int_\Gamma f(x) d\gamma := \int_a^b f(\gamma(t)) | \gamma'(t) |_2 dt$$ \subsubsection*{Kurvenintegral zweiter Art} -Sei vektorwertiges $F \in C(\Gamma, \mathbb{R}^m)$ gegeben: +Sei vektorwertiges $F \in C(\Gamma, \R^m)$ gegeben: \vspace*{-5mm} $$\int_\Gamma F \cdot dx = \int_\Gamma F(x) \cdot dx := \int_a^b (F(\gamma(t))|\gamma'(t)) dt$$ \subsubsection*{Wegunabhängigkeit} -Sei $D \subseteq \mathbb{R}^m$ offen, dann ist $F \in C(D, \mathbb{R}^m)$ wegunabhängig auf $D$, wenn für alle stückweisen $C^1$-Kurven $\gamma_1, \gamma_2 \in C([a, b], \mathbb{R}^m)$ in $D$ mit gleichem Anfangs- und Endpunkt gilt: +Sei $D \subseteq \R^m$ offen, dann ist $F \in C(D, \R^m)$ wegunabhängig auf $D$, wenn für alle stückweisen $C^1$-Kurven $\gamma_1, \gamma_2 \in C([a, b], \R^m)$ in $D$ mit gleichem Anfangs- und Endpunkt gilt: $$\int_{\Gamma_1} F \cdot dx = \int_{\Gamma_2} F \cdot dx$$ -Ein $\phi \in C^1(D, \mathbb{R})$ heißt Potential von $F$ auf $D$, wenn $\nabla\phi = F$ auf $D$. $F$ ist dann Gradientenfeld. +Ein $\phi \in C^1(D, \R)$ heißt Potential von $F$ auf $D$, wenn $\nabla\phi = F$ auf $D$. $F$ ist dann Gradientenfeld. Weiterhin ist $F$ wegunabhängig auf $D$ gdw. $F$ ein Potential $\phi$ auf $D$ hat. @@ -839,13 +839,13 @@ Ferner gilt dann: $\int_\Gamma F \cdot dx = \phi(\gamma(b))-\phi(\gamma(a))$ \subsubsection*{Poincar\'e} -Sei $D \subseteq \mathbb{R}^m$ offen und sternförmig und $F \in C^1(D, \mathbb{R}^m)$ sei rektifizierbar. Dann hat $F$ ein Potential auf $D$, insbesondere auch auf jeder Kugel $B(x_0, r) \subseteq D$. +Sei $D \subseteq \R^m$ offen und sternförmig und $F \in C^1(D, \R^m)$ sei rektifizierbar. Dann hat $F$ ein Potential auf $D$, insbesondere auch auf jeder Kugel $B(x_0, r) \subseteq D$. \section*{Gewöhnliche Differentialgleichung} \subsection*{Lokale Lipschitzstetigkeit} -Sei $M \subseteq \mathbb{R}^m$ und $J$ ein Intervall. $g : J \times M \rightarrow \mathbb{R}^k$ ist lokal Lipschitz in $x$, wenn: +Sei $M \subseteq \R^m$ und $J$ ein Intervall. $g : J \times M \rightarrow \R^k$ ist lokal Lipschitz in $x$, wenn: $\forall (t_0, x_0) \in J \times M \exists \delta = \delta(t_0, x_0) > 0, r = r(t_0, x_0) > 0, L = L(t_0, x_0) \geq 0 \forall t \in [t_0 - \delta, t_0 + \delta] \cap J \forall x, y \in \overline B(x_0, r) \cap M : |g(t,x) - g(t, y)|_2 \leq L|x-y|_2$. @@ -857,7 +857,7 @@ Ein diff. $f$ ist Lipschitz gdw. $\partial_x^{(1)} f$ beschränkt ist. \subsection*{Anfangswertprobleme} -Seien $J \subseteq \mathbb{R}$ ein Intervall, $t_0 \in J$ mit $t_0 < \sup J$, $D \subseteq \mathbb{R}^m$ offen, $f \in C(J \times D, \mathbb{R}^m)$ und $u_0 \in D$. +Seien $J \subseteq \R$ ein Intervall, $t_0 \in J$ mit $t_0 < \sup J$, $D \subseteq \R^m$ offen, $f \in C(J \times D, \R^m)$ und $u_0 \in D$. \vspace*{-4mm} \begin{align*} @@ -865,17 +865,17 @@ Seien $J \subseteq \mathbb{R}$ ein Intervall, $t_0 \in J$ mit $t_0 < \sup J$, $D u(t_0) &= u_0 \end{align*} -Für das Anfangswertproblem wird ein $t_1 \in J$ mit $t_1 > t_0$ und eine eindeutige Lösung $u \in C^1([t_0, t_1], \mathbb{R}^m)$ auf $[t_0, t_1]$ gesucht. +Für das Anfangswertproblem wird ein $t_1 \in J$ mit $t_1 > t_0$ und eine eindeutige Lösung $u \in C^1([t_0, t_1], \R^m)$ auf $[t_0, t_1]$ gesucht. \subsubsection*{Lokale Lipschitzstetigkeit im Kontext} -Sei $f \in C(J \times D, \mathbb{R}^k)$, $D \subseteq \mathbb{R}^m$ offen, $J$ ein Intervall und es existieren alle partiellen Ableitungen $\frac{\partial}{\partial x_j} f \in C(J \times D, \mathbb{R}^k)$ für $j \in \{1, \hdots, m\}$. +Sei $f \in C(J \times D, \R^k)$, $D \subseteq \R^m$ offen, $J$ ein Intervall und es existieren alle partiellen Ableitungen $\frac{\partial}{\partial x_j} f \in C(J \times D, \R^k)$ für $j \in \{1, \hdots, m\}$. Dann ist $f$ lokal Lipschitz in $x$. \subsubsection*{Picard-Lindelöf (lokal)} -Seien $J$ ein Intervall, $D \subseteq \mathbb{R}^m$ offen, $f \in C(J \times D, \mathbb{R}^m)$ lokal Lipschitz in $x$, $u_0 \in D$, $t_0 \in J$ mit $t_0 < \sup J$. Dann gelten: +Seien $J$ ein Intervall, $D \subseteq \R^m$ offen, $f \in C(J \times D, \R^m)$ lokal Lipschitz in $x$, $u_0 \in D$, $t_0 \in J$ mit $t_0 < \sup J$. Dann gelten: \begin{enumerate}[label=(\alph*)] \item $\exists t_1 = t_1(u_0) > t_0 $ mit $t_1 \in J$ und eine eindeutige Lösung $u$ auf $[t_0, t_1]$ von $u'(t) = f(t, u(t))$ mit $u(t_0) = u_0$ @@ -897,11 +897,11 @@ Sei $J$ ein Intervall, $\varphi \in C(J)$, $\varphi \geq 0$, $t_0 = \min J$ und \subsubsection*{Obergrenze des Existenzintervall} -Seien $J = [t_0, \infty)$, $D \subseteq \mathbb{R}^m$ offen, $M \subseteq \mathbb{R}^m$ abgeschlossen mit $M \subseteq D$, $f \in C(J \times D, \mathbb{R}^m)$ sei lokal Lipschitz in $x$, $u$ die maximale Lösung von $u'(t) = f(t, u(t))$ auf $[t_0, t(u_0))$ und es gelte $\forall t_0 \leq t \leq \overline t(u_0) : u(t) \in M$. Dann gelten: +Seien $J = [t_0, \infty)$, $D \subseteq \R^m$ offen, $M \subseteq \R^m$ abgeschlossen mit $M \subseteq D$, $f \in C(J \times D, \R^m)$ sei lokal Lipschitz in $x$, $u$ die maximale Lösung von $u'(t) = f(t, u(t))$ auf $[t_0, t(u_0))$ und es gelte $\forall t_0 \leq t \leq \overline t(u_0) : u(t) \in M$. Dann gelten: \begin{enumerate}[label=(\alph*)] \item $\forall b > t_0 \exists c(b) \geq 0 : \forall t \in [t_0, b], x \in M : (f(t,x)|x) \leq c(b) (1+|x|_2^2) \Rightarrow \overline t(u_0) = \infty$ - \item Sei speziell $f: D \rightarrow \mathbb{R}^n$ und $\exists c \geq 0$ mit $\forall x \in M : (f(x)|x) \leq c(1+|x|_2)$, dann $\overline t(u_0) = \infty$ + \item Sei speziell $f: D \rightarrow \R^n$ und $\exists c \geq 0$ mit $\forall x \in M : (f(x)|x) \leq c(1+|x|_2)$, dann $\overline t(u_0) = \infty$ \end{enumerate} Bedingung (a) folgt aus: $\forall b > t_0 \exists \tilde c(b) \geq 0 \\ \hspace*{4mm} \forall t \in [t_0, b], x \in M : |f(t,x)|_2 \leq \tilde c(b)(1+|x|_2)$ @@ -912,7 +912,7 @@ Bedingung (b) folgt aus: $\exists \tilde c > 0 \forall x \in M : \\ \hspace*{4mm Jedes Anfangswertproblem $k$-ter Ordnung lässt sich in ein Problem 1. Ordnung umschreiben. -Beispielsweise: Das Problem 2. Ordnung $u''(t)=h(t)-u(t)+u'(t)^2$ mit $u(0)=u_0$ und $u'(0)=u_1$ sowie $h \in C(\mathbb{R}, \mathbb{R})$ wird formuliert als Problem 1. Ordnung: +Beispielsweise: Das Problem 2. Ordnung $u''(t)=h(t)-u(t)+u'(t)^2$ mit $u(0)=u_0$ und $u'(0)=u_1$ sowie $h \in C(\R, \R)$ wird formuliert als Problem 1. Ordnung: $$\begin{pmatrix}u(t)\\u'(t)\end{pmatrix}' = \begin{pmatrix}u'(t)\\u''(t)=h(t)-u(t)+u'(t)^2\end{pmatrix}$$ @@ -920,7 +920,7 @@ Sei $v_0(t):=u(t)$, $v_1(t):=u'(t)$ und $v(t):=\begin{pmatrix}v_0(t)\\v_1(t)\end \vspace*{-4mm} \begin{align*} - g : \mathbb{R}^3 &\rightarrow \mathbb{R}^2 \\ + g : \R^3 &\rightarrow \R^2 \\ \begin{pmatrix}t\\v_0\\v_1\end{pmatrix} &\mapsto \begin{pmatrix}v_1\\h(t)-v_0+v_1^2\end{pmatrix} \end{align*} @@ -934,7 +934,7 @@ Insgesamt also: \subsubsection*{Trennung der Variablen} -Sei $u'(t)=g(t)h(u(t))$ mit $u(t_0)=u_0$ Anfangswertproblem mit $g \in C(\mathbb{R}, \mathbb{R})$, $h \in C((a, b), \mathbb{R})$, $u_0 \in (a, b)$ und $h(u_0) \neq 0$. $u$ ist Lösung, wenn $J$ Intervall mit $\forall t \in J : u(t) \in (a, b)$, $u \in C^1(J, \mathbb{R})$ und $t_0 \in J$. +Sei $u'(t)=g(t)h(u(t))$ mit $u(t_0)=u_0$ Anfangswertproblem mit $g \in C(\R, \R)$, $h \in C((a, b), \R)$, $u_0 \in (a, b)$ und $h(u_0) \neq 0$. $u$ ist Lösung, wenn $J$ Intervall mit $\forall t \in J : u(t) \in (a, b)$, $u \in C^1(J, \R)$ und $t_0 \in J$. \vspace*{-5mm} $$u \text{ ist Lösung } \Rightarrow \int_{t_0}^t g(s) ds = \int_{u_0}^{u(t)} \frac{1}{h(x)} dx$$ diff --git a/analysis_3.tex b/analysis_3.tex index d524b8d..483a52f 100644 --- a/analysis_3.tex +++ b/analysis_3.tex @@ -23,18 +23,18 @@ Ein Mengensystem $\mathcal{A} \subseteq \mathcal{P}(X)$ ist $\sigma$-Algebra auf \begin{enumerate}[label=(\alph*)] \item $X \in \mathcal{A}$ \item $A \in \mathcal{A} \Rightarrow A^c := X\setminus A \in \mathcal{A}$ - \item $\forall j \in \mathbb{N} : A_j \in \mathcal{A} \Rightarrow \bigcup_{j\in \mathbb{N}} A_j \in \mathcal{A}$ + \item $\forall j \in \N : A_j \in \mathcal{A} \Rightarrow \bigcup_{j\in \N} A_j \in \mathcal{A}$ \end{enumerate} \subsection*{Eigenschaften von $\sigma$-Algebren} -Seien $\mathcal{A}$ eine $\sigma$-Algebra auf $X$, $n \in \mathbb{N}$, $\forall j \in \mathbb{N} : A_j \in \mathcal{A}$, dann ist $\mathcal{A}$ nach den folgenden Eigenschaften abgeschlossen unter abzählbaren Mengenoperationen: +Seien $\mathcal{A}$ eine $\sigma$-Algebra auf $X$, $n \in \N$, $\forall j \in \N : A_j \in \mathcal{A}$, dann ist $\mathcal{A}$ nach den folgenden Eigenschaften abgeschlossen unter abzählbaren Mengenoperationen: \begin{enumerate}[label=(\alph*)] \item $\emptyset = X^c \in \mathcal{A}$ \item $A_1 \bigcup \cdots \bigcup A_n \in \mathcal{A}$ \item $A_1 \bigcap \cdots \bigcap A_n \in \mathcal{A}$ - \item $\bigcap_{j\in \mathbb{N}} A_j \in \mathcal{A}$ + \item $\bigcap_{j\in \N} A_j \in \mathcal{A}$ \item $A_1 \setminus A_2 := A_1 \bigcap A_2^c \in \mathcal{A}$ \end{enumerate} @@ -62,9 +62,9 @@ Sei $\emptyset \neq \mathcal{E} \subseteq \mathcal{P}(X)$, dann gilt: Sei $X$ ein metrischer Raum und $\mathcal{O}(X)$ das System der in $X$ offenen Mengen, dann ist $\mathcal{B}(X) := \sigma(\mathcal{O}(X))$ die Borelsche $\sigma$-Algebra auf $X$. -Im Speziellen wird $\mathcal{B}_m := \mathcal{B}(\mathbb{R}^m)$ gesetzt. +Im Speziellen wird $\mathcal{B}_m := \mathcal{B}(\R^m)$ gesetzt. -$\mathcal{B}_m$ enthält insb. alle offenen und abgeschlossenen Mengen in $\mathbb{R}^m$ sowie deren abzählbaren Vereinigungen und Durchschnitte. +$\mathcal{B}_m$ enthält insb. alle offenen und abgeschlossenen Mengen in $\R^m$ sowie deren abzählbaren Vereinigungen und Durchschnitte. \subsubsection*{Charakterisierung} @@ -82,7 +82,7 @@ $\mu : \mathcal{A} \rightarrow [0, \infty]$ ist positives Maß auf $\mathcal{A}$ \begin{enumerate}[label=(\alph*)] \item $\mu(\emptyset) = 0$ - \item $\forall \text{ disjunkte } \{A_j | j \in \mathbb{N}\} \subseteq \mathcal{A} :\\ \hspace*{4mm} \mu(\dot\bigcup_{j\in \mathbb{N}} A_j) = \sum_{j\in \mathbb{N}} \mu(A_j)$ + \item $\forall \text{ disjunkte } \{A_j | j \in \N\} \subseteq \mathcal{A} :\\ \hspace*{4mm} \mu(\dot\bigcup_{j\in \N} A_j) = \sum_{j\in \N} \mu(A_j)$ \end{enumerate} \subsection*{Maßraum} @@ -104,21 +104,21 @@ Dieses wird Punkt- / Diracmaß auf $\mathcal{A}$ genannt. \subsection*{Zählmaß} -Sei $\mathcal{A} = \mathcal{P}(\mathbb{N})$ und $\forall j \in \mathbb{N} : p_j \in [0, \infty]$ fest gewählt. +Sei $\mathcal{A} = \mathcal{P}(\N)$ und $\forall j \in \N : p_j \in [0, \infty]$ fest gewählt. -$\mu(A) := \sum_{j\in A} p_j$ für $A \subseteq \mathbb{N}$ ist Maß auf $\mathcal{P}(\mathbb{N})$. +$\mu(A) := \sum_{j\in A} p_j$ für $A \subseteq \N$ ist Maß auf $\mathcal{P}(\N)$. -Gilt zusätzlich $\forall j \in \mathbb{N} : p_j = 1$ so heißt $\mu$ Zählmaß. +Gilt zusätzlich $\forall j \in \N : p_j = 1$ so heißt $\mu$ Zählmaß. \subsection*{Eigenschaften von Maßen} -Sei $(X, \mathcal{A}, \mu)$ Maßraum und $A, B, A_j \in \mathcal{A}$ für $j \in \mathbb{N}$. +Sei $(X, \mathcal{A}, \mu)$ Maßraum und $A, B, A_j \in \mathcal{A}$ für $j \in \N$. \begin{description}[leftmargin=!,labelwidth=26mm] \item[Monotonie] $A \subseteq B \Rightarrow \mu(A) \leq \mu(B)$ - \item[$\sigma$-Subadditivität] $\mu(\dot\bigcup_{j\in \mathbb{N}} A_j) \leq \sum_{j\in \mathbb{N}} \mu(A_j)$ - \item[Stetigkeit (unten)] $A_j \uparrow \Rightarrow \displaystyle\lim_{j\to \infty} \mu(A_j) = \mu(\bigcup_{j\in \mathbb{N}} A_j)$ - \item[Stetigkeit (oben)] $A_j \downarrow \land \hspace*{1mm} \mu(A_1) < \infty \\ \hspace*{4mm}\Rightarrow \displaystyle\lim_{j\to \infty} \mu(A_j) = \mu(\bigcap_{j\in \mathbb{N}} A_j)$ + \item[$\sigma$-Subadditivität] $\mu(\dot\bigcup_{j\in \N} A_j) \leq \sum_{j\in \N} \mu(A_j)$ + \item[Stetigkeit (unten)] $A_j \uparrow \Rightarrow \displaystyle\lim_{j\to \infty} \mu(A_j) = \mu(\bigcup_{j\in \N} A_j)$ + \item[Stetigkeit (oben)] $A_j \downarrow \land \hspace*{1mm} \mu(A_1) < \infty \\ \hspace*{4mm}\Rightarrow \displaystyle\lim_{j\to \infty} \mu(A_j) = \mu(\bigcap_{j\in \N} A_j)$ \end{description} Für $\mu(A) < \infty$ folgt $\mu(B\setminus A) = \mu(B) - \mu(A)$. @@ -131,20 +131,20 @@ Eine Abb. $f : \mathcal{A} \rightarrow [0, \infty)$ ist ein Prämaß auf Ring $\ \begin{enumerate}[label=(\alph*)] \item $\mu(\emptyset) = 0$ - \item $\{A_j | j \in \mathbb{N}\} \subseteq \mathcal{A}$ disjunkt und $A = \bigcup_{j\in \mathbb{N}} A_j \in \mathcal{A} \Rightarrow \mu(A) = \sum_{j\in \mathbb{N}} \mu(A_j)$ + \item $\{A_j | j \in \N\} \subseteq \mathcal{A}$ disjunkt und $A = \bigcup_{j\in \N} A_j \in \mathcal{A} \Rightarrow \mu(A) = \sum_{j\in \N} \mu(A_j)$ \end{enumerate} \section*{Lebesguemaß} \subsection*{System der Intervalle} -Sei $I = (a, b] \subseteq \mathbb{R}^m$ für $a, b \in \mathbb{R}^m$ mit $a \leq b$, dann wird das System von Intervallen $\mathcal{J}_m$ definiert: +Sei $I = (a, b] \subseteq \R^m$ für $a, b \in \R^m$ mit $a \leq b$, dann wird das System von Intervallen $\mathcal{J}_m$ definiert: $\lambda(I) = \lambda_m(I) := (b_1 - a_1) \cdot \hdots \cdot (b_m - a_m)$ \subsection*{Ring der Figuren} -$$\mathcal{F}_m = \left\{ A = \bigcup_{j=1}^n I_j | I_j \in \mathcal{J}_m, n \in \mathbb{N} \right\}$$ +$$\mathcal{F}_m = \left\{ A = \bigcup_{j=1}^n I_j | I_j \in \mathcal{J}_m, n \in \N \right\}$$ \subsubsection*{Eigenschaften} @@ -178,9 +178,9 @@ Seien $\mathcal{A}, \mathcal{B}, \mathcal{C}$ $\sigma$-Algebren auf $X, Y, Z \ne \item $f : X \rightarrow Y$ ist $\mathcal{A}$-$\mathcal{B}$-mb., $g : Y \rightarrow Z$ ist $\mathcal{B}$-$\mathcal{C}$-mb. $\Rightarrow g \circ f : X \rightarrow Z$ ist $\mathcal{A}$-$\mathcal{C}$-mb. \item $\emptyset \neq \mathcal{E} \subseteq \mathcal{P}(Y)$, $\mathcal{B} = \sigma(\mathcal{E})$, $f: X \rightarrow Y$ dann ist $f$ messbar gdw. $\forall E \in \mathcal{E} : f^{-1}(E) \in \mathcal{A}$ \item $X, Y$ metrische Räume, $f : X \rightarrow Y$ stetig $\Rightarrow f$ ist Borel-messbar - \item $f : X \rightarrow \mathbb{R}^m$ ist $\mathcal{A}$-$\mathcal{B}_m$-mb. gdw. $\forall i \in \{1, \dots, m\} : f_i : X \rightarrow \mathbb{R}$ ist $\mathcal{A}$-$\mathcal{B}_1$-mb. - \item $f, g$ sind $\mathcal{A}$-$\mathcal{B}_1$-mb. und $\alpha, \beta \in \mathbb{R} \Rightarrow fg : X \rightarrow \mathbb{R}$ und $\frac{1}{f} : \{x \in X | f(x) \neq 0\} \rightarrow \mathbb{R}$ mb. - \item $f : X \rightarrow \mathbb{R}^m$ ist $\mathcal{A}$-$\mathcal{B}_m$-mb. $\\\Rightarrow g : X \rightarrow \mathbb{R}; x \mapsto |f(x)|_2$ ist $\mathcal{A}$-$\mathcal{B}_1$-mb. + \item $f : X \rightarrow \R^m$ ist $\mathcal{A}$-$\mathcal{B}_m$-mb. gdw. $\forall i \in \{1, \dots, m\} : f_i : X \rightarrow \R$ ist $\mathcal{A}$-$\mathcal{B}_1$-mb. + \item $f, g$ sind $\mathcal{A}$-$\mathcal{B}_1$-mb. und $\alpha, \beta \in \R \Rightarrow fg : X \rightarrow \R$ und $\frac{1}{f} : \{x \in X | f(x) \neq 0\} \rightarrow \R$ mb. + \item $f : X \rightarrow \R^m$ ist $\mathcal{A}$-$\mathcal{B}_m$-mb. $\\\Rightarrow g : X \rightarrow \R; x \mapsto |f(x)|_2$ ist $\mathcal{A}$-$\mathcal{B}_1$-mb. \item $X = W \dot\cup Z$ mit $\emptyset \neq W, Z \in \mathcal{A}$, $f : W \rightarrow Y$ ist $\mathcal{A}_W$-$\mathcal{B}$-mb., $g : Z \rightarrow Y$ ist $\mathcal{A}_Z$-$\mathcal{B}$-mb. $\Rightarrow h(x) = \begin{cases} f(x) & x \in W \\ g(x) & x \in Z diff --git a/lineare_algebra.tex b/lineare_algebra.tex index e503058..a92a3a8 100644 --- a/lineare_algebra.tex +++ b/lineare_algebra.tex @@ -249,7 +249,7 @@ $f \in V^*$ werden als Linearformen bezeichnet. Sei $B = \{b_1, \cdots, b_n\}$ Basis von $V$, dann ist $B^* = \{b_1^*, \cdots, b_n^*\}$ Basis des Dualraums $V^*$, also die zu $B$ von $V$ duale Basis. -Die für $b_i$ eindeutige Abb. $b_i^* : V \rightarrow \mathbb{K}$ erfüllt $b_i^*(b_i) = 1$ und $b_i^*(b_j) = 0$ für $j\neq i$. +Die für $b_i$ eindeutige Abb. $b_i^* : V \rightarrow \K$ erfüllt $b_i^*(b_i) = 1$ und $b_i^*(b_j) = 0$ für $j\neq i$. \subsection*{Faktor- / Quotientenräume} @@ -356,13 +356,13 @@ Reichen die Eigenvektoren nicht aus, können Hauptvektoren hinzugezogen werden. \section*{Skalarprodukte} -\subsection*{Standardskalarprodukt auf $\mathbb{R}^n$} +\subsection*{Standardskalarprodukt auf $\R^n$} -$\langle \cdot, \cdot \rangle : \mathbb{R}^n \times \mathbb{R}^n \rightarrow \mathbb{R}, \langle v, w \rangle := v^T * w$ +$\langle \cdot, \cdot \rangle : \R^n \times \R^n \rightarrow \R, \langle v, w \rangle := v^T * w$ \subsection*{Positive Definitheit} -Eine symmetrische Bilinearform $\langle \cdot, \cdot \rangle : V \times V \rightarrow \mathbb{R}$ ist positiv definit, wenn: +Eine symmetrische Bilinearform $\langle \cdot, \cdot \rangle : V \times V \rightarrow \R$ ist positiv definit, wenn: $\forall v \in V: v \neq 0 \Rightarrow \langle v, v \rangle > 0$ @@ -409,7 +409,7 @@ Eine symmetrische bzw. hermitesche Matrix $A$ ist positiv definit gdw. die Deter \subsection*{Ungleichung von Cauchy-Schwarz} -$\langle v, w \rangle ^2 \leq \langle v, v \rangle * \langle w, w \rangle$ (in $\mathbb{R}$) +$\langle v, w \rangle ^2 \leq \langle v, v \rangle * \langle w, w \rangle$ (in $\R$) $|\langle v, w \rangle |^2 \leq \langle v, v \rangle * \langle w, w \rangle$ (in $\mathbb{C}$) @@ -499,7 +499,7 @@ Unitäre Matrizen sind normal. \subsection*{Iwasawa- / QR-Zerlegung} -Zerlegung von $A \in GL_n(\mathbb{K})$ in das Produkt aus einer orthogonalen bzw. unitären Matrix und einer oberen Dreiecksmatrix. $A = Q \cdot R$. +Zerlegung von $A \in GL_n(\K)$ in das Produkt aus einer orthogonalen bzw. unitären Matrix und einer oberen Dreiecksmatrix. $A = Q \cdot R$. \vspace*{-5mm} $$A = \begin{pmatrix} @@ -536,11 +536,11 @@ Es seien $V, W$ euklidische oder unitäre Vektorräume. Dann ist Isometrie $\phi \subsection*{Eigenwerte von Isometrien} -Seien $\mathbb{K} \in \{\mathbb{R}, \mathbb{C}\}$ und $V$ ein $K$-Vektorraum mit Skalarprodukt, dann: +Seien $\K \in \{\R, \mathbb{C}\}$ und $V$ ein $K$-Vektorraum mit Skalarprodukt, dann: \begin{enumerate}[label=(\alph*)] \item $\phi$ ist lineare Isometrie von $V$, dann $\forall \lambda \in Spec(\phi): |\lambda|=1$ - \item $\alpha \in \mathbb{K}$ mit $|\alpha|=1 \land V \neq \{0\}$, dann gibt es Isometrie von $V$ mit Eigenwert $\alpha$ + \item $\alpha \in \K$ mit $|\alpha|=1 \land V \neq \{0\}$, dann gibt es Isometrie von $V$ mit Eigenwert $\alpha$ \end{enumerate} \subsection*{Isometrien und Orthonormalbasen} @@ -580,7 +580,7 @@ Wobei $D_{\psi_i} = \begin{pmatrix} cos(\psi_i) & -sin(\psi_i) \\ sin(\psi_i) & \section*{Selbstadjungierte Abbildungen} -Sei $V$ Vektorraum mit SKP über $\mathbb{R}$ oder $\mathbb{C}$ und $\phi \in End(V)$. Dann ist $\phi$ selbstadjungiert, wenn für $\forall v, w \in V$ gilt: $\langle \phi(v), w \rangle = \langle v, \phi(w) \rangle$. +Sei $V$ Vektorraum mit SKP über $\R$ oder $\mathbb{C}$ und $\phi \in End(V)$. Dann ist $\phi$ selbstadjungiert, wenn für $\forall v, w \in V$ gilt: $\langle \phi(v), w \rangle = \langle v, \phi(w) \rangle$. $\phi \text{ ist selbstadjungiert} \Leftrightarrow D_{BB}(\phi)=\overline{D_{BB}(\phi)^T}$ @@ -594,28 +594,28 @@ $A^* := \overline{A^T}$, $A = A^* \Leftrightarrow A \text{ ist hermitesch}$ Sei $A \in \mathbb{C}^{n\times n}$, dann: $A^* \cdot A = A \cdot A^*$ -Sei $B \in \mathbb{R}^{n\times n}$, dann: $B^T \cdot B = B \cdot B^T$ +Sei $B \in \R^{n\times n}$, dann: $B^T \cdot B = B \cdot B^T$ Normale Matrizen sind unitär diagonalisierbar. \subsection*{Symmetrische reelle Matrizen} -Eine symmetrische Matrix $A \in \mathbb{R}^{n\times n}$ besitzt ausschließlich reelle Eigenwerte. +Eine symmetrische Matrix $A \in \R^{n\times n}$ besitzt ausschließlich reelle Eigenwerte. Es existiert eine orthogonale Matrix $S \in O(n)$ so, dass $D_A = S^TAS$ eine Diagonalmatrix ist. \subsection*{Spektralsatz} -Sei $V$ Vektorraum über $\mathbb{R}$ oder $\mathbb{C}$ mit SKP und $\phi \in End(V)$. Dann ist äquivalent: +Sei $V$ Vektorraum über $\R$ oder $\mathbb{C}$ mit SKP und $\phi \in End(V)$. Dann ist äquivalent: \begin{enumerate}[label=(\alph*)] \item $\phi$ ist selbstadjungiert - \item Es gibt eine Orthonormalbasis aus Eigenvektoren von $\phi$ und $Spec(\phi) \subset \mathbb{R}$ + \item Es gibt eine Orthonormalbasis aus Eigenvektoren von $\phi$ und $Spec(\phi) \subset \R$ \end{enumerate} \subsubsection*{Positivität} -Eine symmetrische Matrix $A \in \mathbb{R}^{n \times n}$ ist positiv definit gdw. $\forall \lambda \in Spec(A) : \lambda \geq 0$ +Eine symmetrische Matrix $A \in \R^{n \times n}$ ist positiv definit gdw. $\forall \lambda \in Spec(A) : \lambda \geq 0$ \section*{Affine Räume} @@ -643,35 +643,35 @@ Sei $C=(x_1,...x_r,y_1,...y_r)$, dann ergibt die Lösung $z=(-\lambda_1,...,-\la Seien $a, b \in V$, dann ist die affine Gerade durch $a$ und $b$: $\overline{a, b} := \{\lambda a + (1 - \lambda)b | \lambda \in K\} = a + K*(b-a)$ -Für $K = \mathbb{R}$ und $a, b \in V$ wobei $V$ $\mathbb{R}$-Vektorraum: +Für $K = \R$ und $a, b \in V$ wobei $V$ $\R$-Vektorraum: $[a, b] := \{\lambda a + (1 - \lambda)b|0 \leq \lambda \leq 1\}$ (Strecke $\overrightarrow{ab}$) \subsection*{Affine Abbildungen, Affinitäten} -Seien $A$, $B$ affine Räume mit Translationsvektorräumen $V$ und $W$ über $\mathbb{K}$. Abbildung $\phi : A \rightarrow B$ induziert für gewähltes $a \in A$ eine Abbildung $\varphi : V \rightarrow W$ mit $\phi(v+a) = \varphi(v) + \phi(a)$. +Seien $A$, $B$ affine Räume mit Translationsvektorräumen $V$ und $W$ über $\K$. Abbildung $\phi : A \rightarrow B$ induziert für gewähltes $a \in A$ eine Abbildung $\varphi : V \rightarrow W$ mit $\phi(v+a) = \varphi(v) + \phi(a)$. $\phi$ heißt affiner Homomorphismus falls $\varphi$ ein Vektorraumhomomorphismus ist. Invertierbares $\phi$ heißt Affinität. -\subsubsection*{Affiner Standardraum $\mathbb{A}^n(\mathbb{K})$} +\subsubsection*{Affiner Standardraum $\mathbb{A}^n(\K)$} -Alle affinen Selbstabbildungen des affinen Standardraums haben die Gestalt $\phi : A \rightarrow A$ mit $\phi(a) := M \cdot a + t$ für bel. $M \in \mathbb{K}^{n\times n}$ und $t \in \mathbb{K}^n$. +Alle affinen Selbstabbildungen des affinen Standardraums haben die Gestalt $\phi : A \rightarrow A$ mit $\phi(a) := M \cdot a + t$ für bel. $M \in \K^{n\times n}$ und $t \in \K^n$. \subsubsection*{Euklidischer Raum} -Ist $A$ affiner Raum mit $\mathbb{R}$-Vektorraum $V$ als euklidischen Translationsvektorraum, dann ist $A$ ein euklidischer Raum. +Ist $A$ affiner Raum mit $\R$-Vektorraum $V$ als euklidischen Translationsvektorraum, dann ist $A$ ein euklidischer Raum. \subsection*{Quadriken} -Eine Quadrik $Q \subseteq \mathbb{K}^n$ ist $Q := \{ v \in \mathbb{K}^n | F(v) = 0 \}$ wobei $F \in \mathbb{K}[X_1, \cdots, X_n]$ quadratisches Polynom. +Eine Quadrik $Q \subseteq \K^n$ ist $Q := \{ v \in \K^n | F(v) = 0 \}$ wobei $F \in \K[X_1, \cdots, X_n]$ quadratisches Polynom. \subsubsection*{Matrizenform} Das eine Quadrik $Q$ definierende quadratische Polynom lässt sich wie folgt darstellen: -$F(x) = x^TAx + b^Tx + c$ mit $A \in \mathbb{K}^{n\times n}$, $b \in \mathbb{K}^n$ +$F(x) = x^TAx + b^Tx + c$ mit $A \in \K^{n\times n}$, $b \in \K^n$ -Für $char(\mathbb{K})\neq 2$ ist $A$ symmetrisch. +Für $char(\K)\neq 2$ ist $A$ symmetrisch. \subsubsection*{Affine Normalform} diff --git a/numerik_1.tex b/numerik_1.tex index dd7587f..02c8ca5 100644 --- a/numerik_1.tex +++ b/numerik_1.tex @@ -5,7 +5,7 @@ Für $e_{min}, e_{max} \in \mathbb{Z}$, $e_{min} < e_{max}$ ist ein Gleitkommasy \vspace*{-4mm} \begin{align*} \mathcal{F} &= \mathcal{F}(\beta,t,e_{min},e_{max}) \\ - &= \{ \pm m \beta^{e-t} | m \in \mathbb{N}, \beta^{t-1} \leq m \leq \beta^t - 1 \lor m = 0, \\ & \hspace*{16mm}e_{min} \leq e \leq e_{max} \} + &= \{ \pm m \beta^{e-t} | m \in \N, \beta^{t-1} \leq m \leq \beta^t - 1 \lor m = 0, \\ & \hspace*{16mm}e_{min} \leq e \leq e_{max} \} \end{align*} $x \in \mathcal{F} \setminus \{0\} \Rightarrow \beta^{e_{min}-1} \leq |x| \leq \beta^{e_{max}}(1-\beta^{-1})$. @@ -18,7 +18,7 @@ $x=\pm \beta^e ( \frac{d_1}{\beta^1} + \frac{d_2}{\beta^2} + \cdots + \frac{d_t} \subsection*{Relative Maschinengenauigkeit} -$fl(x) \in \mathcal{F}$ ist die $x \in \mathbb{R}$ am nächsten liegende Gleitkommazahl. +$fl(x) \in \mathcal{F}$ ist die $x \in \R$ am nächsten liegende Gleitkommazahl. Für relative Maschinengenauigkeit $\epsilon := \frac{1}{2} \beta^{1-t}$: @@ -43,7 +43,7 @@ Für $\|\Delta x\|_X \rightarrow 0$. Ein Problem $(f, x)$ ist gut konditioniert \subsubsection*{Kondition stetig differenzierbarer Fkt.} -Für $f \in C^1(E, \mathbb{R}^m)$ in Umgebung $E \subseteq \mathbb{R}^n$ von $x$: +Für $f \in C^1(E, \R^m)$ in Umgebung $E \subseteq \R^n$ von $x$: \vspace*{-2mm} $$\kappa_f(x) = \frac{\|f'(x)\| \cdot \|x\|_X}{\|f(x)\|_Y}$$ @@ -52,27 +52,27 @@ $$\kappa_f(x) = \frac{\|f'(x)\| \cdot \|x\|_X}{\|f(x)\|_Y}$$ \subsection*{Induzierte Matrixnorm / Operatornorm} -Für Normen $\| \cdot \|_\circ$, $\| \cdot \|_\star$ auf $\mathbb{K}^n$ bzw. $\mathbb{K}^m$ ist eine Matrixnorm $\| \cdot \| : \mathbb{K}^{m \times n} \rightarrow [0,\infty)$ auf dem Vektorraum der $m \times n$-Matrizen definiert: +Für Normen $\| \cdot \|_\circ$, $\| \cdot \|_\star$ auf $\K^n$ bzw. $\K^m$ ist eine Matrixnorm $\| \cdot \| : \K^{m \times n} \rightarrow [0,\infty)$ auf dem Vektorraum der $m \times n$-Matrizen definiert: \vspace*{-4mm} -$$\|A\| := \max_{v \in \mathbb{K}^n \setminus \{0\}} \frac{\|Av\|_\star}{\|v\|_\circ} = \max_{\{v \in \mathbb{K}^n | \|v\|_\circ = 1 \}} \|Av\|_\star$$ +$$\|A\| := \max_{v \in \K^n \setminus \{0\}} \frac{\|Av\|_\star}{\|v\|_\circ} = \max_{\{v \in \K^n | \|v\|_\circ = 1 \}} \|Av\|_\star$$ \subsubsection*{Eigenschaften} -Für $A \in \mathbb{K}^{m \times n}$ gilt $\forall v \in \mathbb{K}^n : \|Av\|_\star \leq \|A\| \cdot \|v\|_\circ$ +Für $A \in \K^{m \times n}$ gilt $\forall v \in \K^n : \|Av\|_\star \leq \|A\| \cdot \|v\|_\circ$ Submultiplikativität: $\|AB\| \leq \|A\| \cdot \|B\|$ \subsubsection*{Matrix-$p$-Normen} -Induzierte Matrixnorm bei Wahl der $p$-Normen über $\mathbb{K}^n$ bzw. $\mathbb{K}^m$: +Induzierte Matrixnorm bei Wahl der $p$-Normen über $\K^n$ bzw. $\K^m$: \vspace*{-4mm} -$$\|A\|_p := \max_{\{v \in \mathbb{K}^n | \|v\|_p = 1 \}} \|Av\|_p \text{ für } 1 \leq p \leq \infty$$ +$$\|A\|_p := \max_{\{v \in \K^n | \|v\|_p = 1 \}} \|Av\|_p \text{ für } 1 \leq p \leq \infty$$ \subsubsection*{Spaltensummennorm} -Für $A = (a_1, \cdots, a_n)$ mit $a_j \in \mathbb{K}^m$: +Für $A = (a_1, \cdots, a_n)$ mit $a_j \in \K^m$: \vspace*{-4mm} $$\|A\|_1 = \max_{1 \leq j \leq n} \|a_j\|_1 = \max_{1 \leq j \leq n} \sum_{i=1}^m |a_{i,j}|$$ @@ -84,7 +84,7 @@ $$\|A\|_\infty = \max_{1 \leq i \leq m} \sum_{j=1}^n |a_{i,j}|$$ \subsubsection*{Spektralnorm} -Die Matrix-$2$-Norm wird so genannt, da $\|A\|_2 = \sqrt{\lambda_{max}(A^H A)}$ für $\lambda_{max}(A^H A)$ als Bezeichner des größten Eigenwerts von $A^H A \in \mathbb{K}^{n \times n}$. +Die Matrix-$2$-Norm wird so genannt, da $\|A\|_2 = \sqrt{\lambda_{max}(A^H A)}$ für $\lambda_{max}(A^H A)$ als Bezeichner des größten Eigenwerts von $A^H A \in \K^{n \times n}$. $\|A\|_2 = \|A^H\|_2$, $\|A^H A\|_2 = \|A\|_2^2$ @@ -92,7 +92,7 @@ $\|Q A\|_2 = \|A\|_2$ für unitäre $Q$. \subsection*{Kondition einer Matrix} -Für $A \in \mathbb{K}^{n \times n} \in GL_n{\mathbb{R}}$, $\|\cdot\|$ induzierte Matrixnorm: +Für $A \in \K^{n \times n} \in GL_n{\R}$, $\|\cdot\|$ induzierte Matrixnorm: \vspace*{-4mm} \begin{align*} @@ -104,7 +104,7 @@ Für $A \in \mathbb{K}^{n \times n} \in GL_n{\mathbb{R}}$, $\|\cdot\|$ induziert \subsection*{Cramersche Regel} -Sei $A = (a_{i,j})_{ij} \in GL_n(\mathbb{R})$, $b \in \mathbb{R}^n$, $A[j] = (a_1, \cdots, a_{j-1}, b, a_{j+1}, \cdots, a_n) \in \mathbb{R}^{n \times n}$, $a_k$ k-ter Spaltenvektor von $A$. Dann bildet $x_j = \frac{det(A[j])}{det(A)}$ für $j = 1, \cdots, n$ die eindeutige Lösung $x \in \mathbb{R}^n$ s.d. $Ax=b$. +Sei $A = (a_{i,j})_{ij} \in GL_n(\R)$, $b \in \R^n$, $A[j] = (a_1, \cdots, a_{j-1}, b, a_{j+1}, \cdots, a_n) \in \R^{n \times n}$, $a_k$ k-ter Spaltenvektor von $A$. Dann bildet $x_j = \frac{det(A[j])}{det(A)}$ für $j = 1, \cdots, n$ die eindeutige Lösung $x \in \R^n$ s.d. $Ax=b$. Aufgrund des hohen Aufwands von allg. mehr als $(n+1)!$ arithmetischen Operationen ist die Cramersche Regel nur von theoretischer Bedeutung. diff --git a/zusammenfassung.tex b/zusammenfassung.tex index 57f5408..d6c4ae9 100644 --- a/zusammenfassung.tex +++ b/zusammenfassung.tex @@ -16,6 +16,10 @@ \setlength{\parindent}{0mm} +\newcommand{\R}{\mathbb{R}} +\newcommand{\N}{\mathbb{N}} +\newcommand{\K}{\mathbb{K}} + \begin{document} \small{ \begin{multicols*}{4} -- cgit v1.2.3